Exercice n°3

Partie

Calculer les intégrales indéfinies suivantes (on indiquera l'ensemble de définition):

Question

\(\displaystyle{\int\frac{x^3-1}{x^3+1}dx}\)

Solution détaillée

Première étape : décomposition de la fraction rationnelle

0n a \(\displaystyle{X^3+1=(X+1)(X^2-X+1)}\),

la décomposition est donc de la forme :

\(\displaystyle{\frac{X^3-1}{X^3+1}=1+\frac{a}{(X+1)}+\frac{bX+c}{X^2-X+1}}\).

On considère la fonction rationnelle \(\displaystyle{x\to\frac{x^3-1}{x^3+1}=1+\frac{a}{x+1}+\frac{bx+c}{x^2-x+1}}\)définie sur les intervalles \(\displaystyle{]-\infty,-1[\textrm{ et }]-1,+\infty[}\).

On détermine les coefficients avec les opérations suivantes :

- on multiplie par \(x+ 1\) les deux membres de l'égalité et on prend la valeur en \(x = -1\), on détermine ainsi \(a\)

- puis on multiplie par \(x\) les deux membres de l'égalité et on prend la limite pour \(x\) tendant vers \(+\infty\) ,

- enfin on prend une valeur particulière \(x = 0\), par exemple.

Ces deux dernières opérations permettent de déterminer \(b\) et \(c\)

On obtient alors :

\(\displaystyle{\frac{X^3-1}{X^3+1}=1-\frac{2}{3(X+1)}+\frac{2}{3}\frac{X-2}{(X^2-X+1)}}\) .

Seconde étape : intégration

On a sur l'intervalle \(]-1,+\infty[\)

\(\displaystyle{\int\frac{x^3-1}{x^3+1}dx=x-\frac{2}{3}\ln(x+1)+\frac{2}{3}\int\frac{x-2}{x^2-x+1}dx}\)

Or on a :

\(\displaystyle{\frac{2}{3}\int\frac{x-2}{x^2-x+1}dx=\frac{1}{3}\int\frac{2x-1}{x^2-x+1}dx-\frac{dx}{x^2-x+1}}\) .

Reste donc à calculer l'intégrale indéfinie \(\displaystyle{\int\frac{dx}{x^2-x+1}}\).

Le trinôme mis sous forme canonique : \(\displaystyle{x^2-x+1=(x-\frac{1}{2})^2+\frac{3}{4}}\), le changement de variable défini par

\(\displaystyle{x-\frac{1}{2}=\frac{\sqrt{3}}{2}u}\), conduit alors à l'intégration

\(\displaystyle{\frac{2}{\sqrt{3}}\int\frac{du}{u^2+1}=\frac{2}{\sqrt{3}}\arctan u}\)

d'où sur l'intervalle \(]-1,+\infty[\)

\(\displaystyle{\int\frac{x^3-1}{x^3+1}dx=x-\frac{2}{3}\ln(x+1)+\frac{1}{3}\ln(x^2-x+1)-\frac{2\sqrt{3}}{3}\arctan(\frac{2x-1}{\sqrt{3}})+k}\)

Pour obtenir l'expression sur \(]-\infty,-1[\), il suffit de remplacer \(\ln( x+ 1)\) par \(\ln( -x- 1)\).

Question

\(\displaystyle{\int\frac{x^2+25x-1}{(x-1)(x+1)(x^2+4)^2}}\)

Solution détaillée

La stratégie classique est ici à éviter :en effet la présence du facteur \((X^2+4)^2\) au dénominateur conduit à une décomposition de la fraction rationnelle qui comporte des éléments de la forme :

\(\displaystyle{\frac{aX+b}{(X^2+4)^2}+\frac{cX+d}{X^2+4}}\).

Quatre coefficients à déterminer c'est beaucoup ! Les ressources habituelles \(\displaystyle{(x=0,x\to+\infty)}\) sont insuffisantes. On peut alors songer à deux stratégies possibles pour la décomposition qu'on écrit de façon indéterminée

\(\displaystyle{\frac{X^2+25X-1}{(X+1)(X-1)(X^2+4)^2}=\frac{a}{X+1}+\frac{b}{X-1}+\frac{cX+d}{(X^2+4)^2}+\frac{\textrm{e}X+f}{(X^2+4)}}\)

Stratégie semi-classique

On détermine immédiatement les coefficients \(a\) et \(b\) qui valent \(1/2\). On forme alors

\(\displaystyle{\frac{X^2+25X-1}{(X+1)(X-1)(X^2+4)^2}-\frac{1}{2}(\frac{1}{X+1}+\frac{1}{X-1})=\frac{-X^3-9x+1}{(X^2+4)^2}=\frac{-X(X^2+4)-5X+1}{(X^2+4)^2}}\)

\(\displaystyle{=-\frac{X}{X^2+4}+\frac{-5X+1}{(X^2+4)^2}}\).

On a alors, par exemple, sur l'intervalle \(]1,+\infty[\) (ou \(-\infty,-1\))

\(\displaystyle{\int(\frac{1}{2}(\frac{1}{x+1}+\frac{1}{x-1})-\frac{x}{x^2+4}-5\frac{x}{(x^2+4)^2})dx=\frac{1}{2}(\ln(x^2-1)-\ln(x^2+4))+\frac{5}{2(x^2+4)}+k}\)  Il reste alors à calculer \(I(x)=\int\frac{dx}{(x^2+4)^2}\).  On procède, comme il a été indiqué dans le cours, en faisant le changement de variable \(u =2x\), d'où

\(\displaystyle{8I(x)=\int\frac{du}{(u^2+1)^2}=\int(\frac{u^2+1}{(u^2+1)^2}-\frac{u^2}{(u^2+1)^2})du}\)

 Une intégration par parties donne alors

\(\displaystyle{I(x)=\frac{1}{16}\arctan\frac{x}{2}+\frac{x}{8(x^2+4)}}\)

 On a donc finalement :

sur les intervalles \(]-\infty,-1[\) et \(]1,+\infty[\)

\(\displaystyle{\int\frac{x^2+25x-1}{(x-1)(x+1)(x^2+4)^2}dx=\ln\sqrt{\frac{x^2-1}{x^2+4}}+\frac{x+20}{8(x^2+4)}+\frac{1}{16}\arctan(\frac{x}{2})+k}\)

sur l'intervalle \(]-1,+1[\) .

\(\displaystyle{\int\frac{x^2+25x-1}{(x-1)(x+1)(x^2+4)^2}dx=\ln\sqrt{\frac{1-x^2}{x^2+4}}+\frac{x+20}{8(x^2+4)}+\frac{1}{16}\arctan(\frac{x}{2})+k}\)

Stratégie “astuce“

On écrit

\(\displaystyle{\frac{X^2-1+25X}{(X^2-1)(X^2+4)^2}=\frac{1}{(X^2+4)^2}+\frac{25X}{(X^2-1)(X^2+4)^2}}\)

Le premier terme conduit à intégrer \(\displaystyle{\int\frac{dx}{(x^2+4)^2}}\) et le second avec le changement de variable \(u= x^2\)

\(\displaystyle{25\int\frac{du}{(u-1)(u+4)^2}}\)

On termine l'intégration comme précédemment .